- Fri Jan 21, 2011 12:00 am
#22890
Complete Question Explanation
Strengthen. The correct answer choice is (B)
The initial premise is that when a business has linked computer systems with identical operating systems, a vandal who accesses one computer has access to the data on all of the company's computers.
The vandal can then introduce a "virus" to destroy data.
The stimulus then establishes that minor variations in operating systems software could virtually eliminate the vandal's ability to access the computers all at once, and that the computers would still be compatible enough for business purposes.
The argument concludes that businesses should implement minor variations in the operating system software of its various computers.
You should notice that the stimulus has already covered the most likely commonsense objection to varying operating systems by pointing out that compatibility is not an issue. However, it is still entirely possible that there are concerns other than compatibility that could affect the argument. There could be other solutions to the problem, or the proposed solution might be overly costly, or inefficient in some way. Since you are asked to strengthen the argument, it is likely that you will simply choose an answer that eliminates a concern, which is the Defender model for assumption and strengthen answers.
Answer choice (A): This choice seeks to establish that it is easy for a vandal to gain access across businesses. However, the stimulus concerned access within a single business, so this choice is incorrect. If you thought that this choice made it more likely that businesses were at risk without varying their operating software, you should realize that answer choice (A) only refers to an increase in compatibility, and does not infer that the systems are similar enough for a multi-business attack. Furthermore, the stimulus was concerned with how to deal with the existing risk. Proving that the risk is even greater does not strengthen the idea that the solution proposed in the stimulus is correct.
Answer choice (B): This is the correct answer choice. If it were true that it would be cheaper to simply repair the damage from attacks than to replace our operating systems, it might not make sense to replace our systems. Since this choice points out that repairs following an attack would be more expensive than preventing the attack, answer choice (B) eliminates the concern that the preventative measures might be more costly than simply enduring the problem.
Answer choice (C): This choice is entirely irrelevant. The LSAT test writers have included this choice on the possibility that you missed the premise that eliminated compatibility as an issue. If you chose this answer, you probably need to be more careful when reading the stimulus.
Answer choice (D): This choice could actually suggest that there is no way to defend against attacks in general, thereby challenging the conclusion. You should eliminate this choice, as it either is irrelevant or weakens the argument.
Answer choice (E): This choice is irrelevant. Just as in answer choice (C), this response is for those who fell prey to a careless reading, and neglected that compatibility is a non-issue.
Strengthen. The correct answer choice is (B)
The initial premise is that when a business has linked computer systems with identical operating systems, a vandal who accesses one computer has access to the data on all of the company's computers.
The vandal can then introduce a "virus" to destroy data.
The stimulus then establishes that minor variations in operating systems software could virtually eliminate the vandal's ability to access the computers all at once, and that the computers would still be compatible enough for business purposes.
The argument concludes that businesses should implement minor variations in the operating system software of its various computers.
You should notice that the stimulus has already covered the most likely commonsense objection to varying operating systems by pointing out that compatibility is not an issue. However, it is still entirely possible that there are concerns other than compatibility that could affect the argument. There could be other solutions to the problem, or the proposed solution might be overly costly, or inefficient in some way. Since you are asked to strengthen the argument, it is likely that you will simply choose an answer that eliminates a concern, which is the Defender model for assumption and strengthen answers.
Answer choice (A): This choice seeks to establish that it is easy for a vandal to gain access across businesses. However, the stimulus concerned access within a single business, so this choice is incorrect. If you thought that this choice made it more likely that businesses were at risk without varying their operating software, you should realize that answer choice (A) only refers to an increase in compatibility, and does not infer that the systems are similar enough for a multi-business attack. Furthermore, the stimulus was concerned with how to deal with the existing risk. Proving that the risk is even greater does not strengthen the idea that the solution proposed in the stimulus is correct.
Answer choice (B): This is the correct answer choice. If it were true that it would be cheaper to simply repair the damage from attacks than to replace our operating systems, it might not make sense to replace our systems. Since this choice points out that repairs following an attack would be more expensive than preventing the attack, answer choice (B) eliminates the concern that the preventative measures might be more costly than simply enduring the problem.
Answer choice (C): This choice is entirely irrelevant. The LSAT test writers have included this choice on the possibility that you missed the premise that eliminated compatibility as an issue. If you chose this answer, you probably need to be more careful when reading the stimulus.
Answer choice (D): This choice could actually suggest that there is no way to defend against attacks in general, thereby challenging the conclusion. You should eliminate this choice, as it either is irrelevant or weakens the argument.
Answer choice (E): This choice is irrelevant. Just as in answer choice (C), this response is for those who fell prey to a careless reading, and neglected that compatibility is a non-issue.